From 2987081fa75539cd4a45b632a264d2c320ec0698 Mon Sep 17 00:00:00 2001 From: "W. Trevor King" Date: Wed, 25 Jul 2012 08:40:44 -0400 Subject: [PATCH 1/1] Fix subscripting (r_12 -> r_{12}) in Serway and Jewett v8's 25.21. --- latex/problems/Serway_and_Jewett_8/problem25.21.tex | 2 +- 1 file changed, 1 insertion(+), 1 deletion(-) diff --git a/latex/problems/Serway_and_Jewett_8/problem25.21.tex b/latex/problems/Serway_and_Jewett_8/problem25.21.tex index 116769b..641ddd4 100644 --- a/latex/problems/Serway_and_Jewett_8/problem25.21.tex +++ b/latex/problems/Serway_and_Jewett_8/problem25.21.tex @@ -11,7 +11,7 @@ on the $y$ axis at $y=0.500\U{m}$. \begin{solution} \Part{a} \begin{equation} - V_3 = k_e \frac{q_1}{r_13} + k_e \frac{q_2}{r_12} + V_3 = k_e \frac{q_1}{r_{13}} + k_e \frac{q_2}{r_{12}} = 2 k_e \frac{2\U{$\mu$C}}{\sqrt{(1.00\U{m})^2 + (0.500\U{m})^2}} = \ans{32.2\U{kV}} \end{equation} -- 2.26.2